0
$\begingroup$

I am trying to compute the weigth of an ellipse with the density: $\exp(-r^2/2)$ or:

$M_2=\int_{U} \frac{1}{(2\pi)}\exp\Big(-\frac{(x^2+y^2)}{2}\Big)\; dx dy \quad\text{et}\quad U=\Big\{ \frac{x^2}{a^2}+\frac{y^2}{b^2}<1\Big\}$ and more generally:

$M_n=\int_{U}\frac{1}{{(2\pi)}^{n/2}}\exp\Big(-\frac{1}{2}\Big(x_1^2+\cdots+x_n^2\Big)\Big)\; dx_1\cdots dx_n\quad\text{et}\quad U=\{ \frac{x_1^2}{a_1^2}+\cdots+\frac{x_n^2}{a_n^2}<1\}$

Simple variable substitutions (polar, rational) don't work. I believe that one needs use improper convergent integrals and use the residue theorem. This seems to be a delicat matter.

Thanks for your help.

JP

$\endgroup$
2
  • 7
    $\begingroup$ These are not "elliptic integrals"... elliptic integrals arise from the calculation of arc lengths of an ellipse. See: en.wikipedia.org/wiki/Elliptic_integral $\endgroup$ Feb 14, 2018 at 13:22
  • 2
    $\begingroup$ I took the liberty of changing the title in response to this comment. $\endgroup$ Feb 14, 2018 at 20:00

1 Answer 1

7
$\begingroup$

The integral $M_2$ was studied in a 1961 US military report, to calculate the probabability of a missile hitting an elliptical target. See equation 31, where the integral $M_2=p(a,b)$ is given as a sum over Bessel functions $I_n$. (There does not seem to be a closed form solution.)

The answer can be seen as a two-dimensional analogue of the error-function, defined as an integral over $I_0$,

$$E(R,r)=e^{-r^2/2}\int_0^R e^{-t^2/2}I_0(rt)tdt$$

Then the desired integral over the ellipse is

$$M_2=\frac{1}{2\pi}\int\int_{x^2/a^2+y^2/b^2\leq 1} e^{-(x^2+y^2)/2}dxdy$$ $$=E[(a+b)/2,(a-b)/2]-E[(a-b)/2,(a+b)/2]$$

$\endgroup$
3
  • $\begingroup$ Thank you very much Carlo for this information and solution. It does not bode well for a simple solution in the case n>2. $\endgroup$
    – JP M
    Feb 19, 2018 at 10:34
  • $\begingroup$ I tried to calculate E(R,r) with[I_0(x)=\frac{1}{\pi}\int_0^\pi \cos(-x\sin \theta)d\theta=\frac{1}{2\pi}\int_{-\pi}^{\pi} e^{ix\sin \theta}d\theta] $\endgroup$
    – JP M
    Feb 19, 2018 at 10:36
  • $\begingroup$ but did not get anywhere. JP $\endgroup$
    – JP M
    Feb 19, 2018 at 10:37

Your Answer

By clicking “Post Your Answer”, you agree to our terms of service and acknowledge you have read our privacy policy.

Not the answer you're looking for? Browse other questions tagged or ask your own question.